Tải bản đầy đủ (.pdf) (60 trang)

BẤT ĐẲNG THỨCTRONG LỚP HÀM SIÊU VIỆT

Bạn đang xem bản rút gọn của tài liệu. Xem và tải ngay bản đầy đủ của tài liệu tại đây (328.67 KB, 60 trang )

ĐẠI HỌC QUỐC GIA HÀ NỘI
TRƯỜNG ĐẠI HỌC KHOA HỌC TỰ NHIÊN
-----------------------

NGUYỄN THỊ HỒNG DUYÊN

BẤT ĐẲNG THỨC
TRONG LỚP HÀM SIÊU VIỆT

Chuyên ngành: PHƯƠNG PHÁP TOÁN SƠ CẤP
Mã số:
60.46.01.13

LUẬN VĂN THẠC SỸ KHOA HỌC

NGƯỜI HƯỚNG DẪN KHOA HỌC:
GS. TSKH. NGUYỄN VĂN MẬU

Hà Nội - Năm 2015


1

Mục lục
Mở đầu

2

1 Một số tính chất của hàm mũ và logarit
1.1 Hàm đơn điệu . . . . . . . . . . . . . . . . . . . . . . . . . .
1.2 Hàm lồi, lõm . . . . . . . . . . . . . . . . . . . . . . . . . . .


1.3 Tính đơn điệu, tính lồi lõm của hàm số mũ và hàm logarit . .
1.3.1 Tính đơn điệu của hàm số mũ và hàm logarit . . . . .
1.3.2 Tính lồi, lõm của hàm số mũ và hàm logarit . . . . . .
1.4 Một số bất đẳng thức cổ điển . . . . . . . . . . . . . . . . . .
1.5 Vai trò của hàm số mũ, hàm logarit trong chứng minh các bất
đẳng thức cổ điển . . . . . . . . . . . . . . . . . . . . . . . .

4
4
5
5
5
6
6
9

2 Các bất đẳng thức trong lớp hàm mũ, hàm logarit
13
2.1 Bất đẳng thức hàm số mũ . . . . . . . . . . . . . . . . . . . 13
2.2 Bất đẳng thức hàm logarit . . . . . . . . . . . . . . . . . . . 26
3 Một số bài toán áp dụng
3.1 Các bài toán cực trị liên quan đến hàm mũ và hàm logarit . .
3.2 Bất đẳng thức siêu việt trong dãy số và giới hạn . . . . . . .
3.3 Bất đẳng thức siêu việt trong phương trình và hệ phương trình

34
34
42
50


KẾT LUẬN

58

Tài liệu tham khảo

59


2

Mở đầu
Bất đẳng thức là một lĩnh vực khó trong chương trình toán học phổ
thông, song nó lại luôn có sức hấp dẫn, thu hút sự tìm tòi, óc sáng tạo của
học sinh. Dạng toán về bất đẳng thức thường có mặt trong các kỳ thi tuyển
sinh cao đẳng đại học, thi học sinh giỏi hay các kỳ thi Olympic. Lý thuyết
bất đẳng thức và đặc biệt, các bài tập về bất đẳng thức rất phong phú và
cực kỳ đa dạng. Đặc biệt bất đẳng thức trong lớp hàm siêu việt là một phần
chuyên đề rất hay, đóng vai trò quan trọng trong bồi dưỡng học sinh giỏi.
Để góp phần đáp ứng nhu cầu bồi dưỡng giáo viên và bồi dưỡng học
sinh giỏi về bất đẳng thức, luận văn "Bất đẳng thức trong lớp hàm siêu việt"
đưa ra một số bài toán bất đẳng thức trong lớp hàm mũ và logarit, một số
bài toán áp dụng cúa bất đẳng thức siêu việt vào việc tìm giá trị lớn nhất,
giá trị nhỏ nhất của hàm số, các bài toán dãy số và giới hạn và khảo sát
một số phương trình và hệ phương trình. Luận văn "Bất đẳng thức trong
lớp hàm siêu việt" chủ yếu là sưu tầm, nghiên cứu tài liệu và các sách tham
khảo liên quan đến bất đẳng thức trong lớp hàm mũ, logarit và các bài toán
ứng dụng liên quan. Luận văn là một chuyên đề nhằm góp phần hướng tới
bồi dưỡng học sinh giỏi bậc trung học phổ thông (xem [1-9]).
Ngoài phần Mở đầu và Kết luận, Luận văn được chia làm ba chương

như sau:
Chương 1. Các kiến thức bổ trợ.
Chương này trình bày một số tính chất của hàm số mũ và hàm logarit
(tính đơn điệu, tính lồi lõm); ý nghĩa của hàm số mũ, hàm logarit trong
chứng minh các bất đẳng thức cổ điển và một số bất đẳng thức cổ điển được
sử dụng trong luận văn.


3

Chương 2. Các bất đẳng thức trong lớp hàm mũ, logarit.
Chương này đưa ra các bài toán về bất đẳng thức mũ, logarit được
nghiên cứu và tổng hợp trong các tài liệu tham khảo.
Chương 3. Một số bài toán áp dụng.
Chương này đưa ra các bài toán cực trị liên quan đến hàm mũ, hàm
logarit; các bài toán áp dụng bất đẳng thức siêu việt trong dãy số và giới
hạn, trong khảo sát phương trình và hệ phương trình.
Trong thời gian thực hiện luận văn này, tác giả đã nhận được sự hướng
dẫn, chỉ bảo tận tình của GS. TSKH. Nguyễn Văn Mậu. Thông qua luận
văn này, tác giả xin được bày tỏ lòng biết ơn sâu sắc và trân trọng những
công lao, sự quan tâm, động viên và sự tận tình chỉ bảo, hướng dẫn của thầy
Nguyễn Văn Mậu.
Tác giả chân thành cảm ơn các thầy giáo, cô giáo khoa Toán - Cơ - Tin
học đã dạy bảo tận tình; chân thành cảm ơn các thầy cô trong Ban giám
hiệu, Phòng đào tạo Sau đại học, văn phòng khoa Toán - Cơ - Tin trường
Đại học Khoa học Tự nhiên - Đại học Quốc gia Hà Nội đã tạo mọi điều kiện
thuận lợi trong suốt thời gian tác giả học tập và làm luận văn.

Hà Nội, ngày 30 tháng 11 năm 2015
Học viên

Nguyễn Thị Hồng Duyên


4

Chương 1
Một số tính chất của hàm mũ và
logarit
1.1

Hàm đơn điệu

Định nghĩa 1.1 (Xem [1-3]). Cho hàm số f : R → R xác định trên tập

I(a; b) ⊂ R, trong đó I(a, b) là ký hiệu một trong các tập hợp (a, b), [a, b), (a, b], [a, b]
với a < b. Khi đó, nếu ứng với mọi x1 , x2 ∈ I(a, b), ta đều có với x1 < x2
suy ra f (x1 ) ≤ f (x2 ) thì ta nói rằng f (x) là hàm đơn điệu tăng trên I(a; b).
Đặc biệt, khi ứng với mọi cặp x1 , x2 ∈ I(a; b), ta đều có

f (x1 ) < f (x2 ) ⇔ x1 < x2
thì ta nói rằng f (x) là một hàm đơn điệu tăng thực sự trên I(a; b), hay còn
gọi là hàm đồng biến.
Ngược lại, nếu ứng với mọi x1 , x2 ∈ I(a, b), ta đều có với x1 < x2 suy
ra f (x1 ) ≥ f (x2 ) thì ta nói rằng f (x) là hàm đơn điệu giảm trên I(a; b).
Nếu

f (x1 ) > f (x2 ) ⇔ x1 < x2 ; ∀x1 , x2 ∈ I(a; b)
thì ta nói rằng f (x) là một hàm đơn điệu giảm thực sự trên I(a; b), hay còn
gọi là hàm nghịch biến.
Định lý 1.1. Giả sử hàm số f (x) có đạo hàm trên khoảng (a; b) và f (x) > 0

với mọi x ∈ (a; b) thì hàm số f (x) đồng biến trên khoảng đó. Ngược lại, nếu

f (x) < 0 với mọi x ∈ (a; b) thì hàm số f (x) nghịch biến trên khoảng đó.


5

1.2

Hàm lồi, lõm

Định nghĩa 1.2 (Xem [1-3]). Hàm số f (x) được gọi là hàm lồi (lồi xuống
dưới) trên tập I(a; b) ⊂ R nếu với mọi x1 , x2 ∈ I(a; b) và với mọi cặp số
dương α, β có tổng α + β = 1, ta đều có

f (αx1 + βx2 ) ≤ αf (x1 ) + βf (x2 ).
Nếu dấu đẳng thức xảy ra khi và chỉ khi x1 = x2 ta nói hàm số f (x) là hàm
lồi thực sự (chặt) trên I(a; b).
Hàm số f (x) được gọi là hàm lõm (lồi trên) trên tập I(a; b) ⊂ R nếu với mọi

x1 , x2 ∈ I(a; b) và với mọi cặp số dương α, β có tổng α + β = 1, ta đều có
f (αx1 + βx2 ) ≥ αf (x1 ) + βf (x2 ).
Nếu dấu đẳng thức xảy ra khi và chỉ khi x1 = x2 ta nói hàm số f (x) là hàm
lõm thực sự (chặt) trên I(a; b)
Định lý 1.2 (Xem [1-3]). Nếu f (x) khả vi bậc hai trên I(a; b) thì f (x) lồi
(lõm) trên I(a; b) khi và chỉ khi f (x) ≥ 0 (f (x) ≤ 0) trên I(a; b).

1.3

Tính đơn điệu, tính lồi lõm của hàm số mũ và

hàm logarit

1.3.1

Tính đơn điệu của hàm số mũ và hàm logarit

- Xét hàm số y = ax , a > 0, a = 1 liên tục trên R, ta có

y = ax ln a (a > 0, a = 1).
Khi a > 1 thì y > 0 nên hàm số đồng biến trên R.
Khi 0 < a < 1 thì y < 0 nên hàm số nghịch biến trên R.
- Xét hàm số y = loga x, a > 0, a = 1; x > 0 ta có

y = (loga x) =

1
.
x · ln a

Khi a > 1 thì y > 0 nên hàm số đồng biến trên (0; +∞).
Khi 0 < a < 1 thì y < 0 nên hàm số nghịch biến trên (0; +∞).


6

1.3.2

Tính lồi, lõm của hàm số mũ và hàm logarit

- Xét hàm số y = ax , a > 0, a = 1, ta có


y = ax ln a (a > 0, a = 1),
y = (ln a)2 ax .
Ta thấy y > 0 với mọi 0 < a = 1, x ∈ R do đó hàm số y = ax là hàm lồi
trên R.
- Tương tự, với hàm số y = loga x, a > 0, a = 1; x > 0, ta có
1
.
y = (loga x) =
x · ln a
−1
y = 2
.
x ln a
Nếu a > 1 tức ln a > 0 thì y < 0 suy ra hàm số lõm trên (0; +∞).
Nếu 0 < a < 1 tức ln a < 0 thì y > 0 suy ra hàm số lồi trên (0; +∞).

1.4

Một số bất đẳng thức cổ điển

Định lý 1.3 (Bất đẳng thức AM - GM, Xem [1-3]). Giả sử x1 , x2 , . . . , xn là
các số không âm. Khi đó

x1 + x2 + · · · + xn
≥ n x1 x2 . . . xn .
n
Dấu đẳng thức xảy ra khi và chỉ khi x1 = x2 = · · · = xn .
Định lý 1.4 (Bất đẳng thức dạng Karamata, Xem [1]). Cho hai dãy số


xk , yk ∈ I(a, b), k = 1, 2, . . . , n, thỏa mãn các điều kiện
x1 ≥ x2 ≥ · · · ≥ xn , y1 ≥ y2 ≥ · · · ≥ yn

x1 ≥ y1 ,




x1 + x2 ≥ y1 + y2 ,
· · · · · · ··


x1 + x2 + · · · + xn−1 ≥ y1 + y2 + · · · + yn−1 ,



x1 + x2 + · · · + xn = y1 + y2 + · · · + yn .
Khi đó, ứng với mọi hàm lồi thực sự f (x) (f (x) > 0) trên I(a, b), ta đều có

f (x1 ) + f (x2 ) + · · · + f (xn ) ≥ f (y1 ) + f (y2 ) + · · · + f (yn ).


7

Định lý 1.5 (Bất đẳng thức Jensen, Xem [1]). Cho hàm số y = f (x) liên
tục và lồi trên [a, b]. Cho các số k1 , k2 , . . . , kn ∈ R+ ; k1 + k2 + · · · + kn = 1.
Khi đó với mọi xi ∈ [a, b]; i = 1, 2, . . . , n, ta luôn có
n

n


ki f (xi ) ≥ f (
i=1

ki xi ).
i=1

Nếu hàm số y = f (x) lõm trên [a, b] thì bất đẳng thức trên đổi chiều, tức là
n

n

ki f (xi ) ≤ f (
i=1

ki xi ).
i=1

Định lý 1.6 (Bất đẳng thức Bernoulli (dạng liên tục), Xem [1]). Cho x > 0.
Khi đó

xα + (1 − x)α ≥ 1

Khi α ≥ 1 hoặc α ≤ 0.

xα + (1 − x)α ≤ 1

Khi 0 ≤ α ≤ 1.

Định lý 1.7 (Bất đẳng thức Bernoulli đối với tam thức bậc (α, β) , Xem [1]

). Cho cặp số (α, β) thỏa mãn điều kiện α > β > 0. Khi đó, với mọi x ∈ R+

xα +

α
α
− 1 ≥ xβ .
β
β

Dấu đẳng thức xảy ra khi và chỉ khi x = 1.

Định lý 1.8 (Bất đẳng thức Schur). Với các số thực dương a, b, c và k ∈ R+
bất kỳ ta luôn có

ak (a − b)(a − c) + bk (b − c)(b − a) + ck (c − a)(c − b) ≥ 0.
Dấu đẳng thức xảy ra khi và chỉ khi a = b = c hoặc a = b và c = 0 cùng các
hoán vị của nó.
Hai trường hợp quen thuộc được sử dụng nhiều là k = 1 và k = 2 tức là
(i) a(a − b)(a − c) + b(b − c)(b − a) + c(c − a)(c − b) ≥ 0.
(ii) a2 (a − b)(a − c) + b2 (b − c)(b − a) + c2 (c − a)(c − b) ≥ 0.


8

Phương pháp đổi biến p, q, r
Đối với một số bài bất đẳng thức thuần nhất đối xứng có các biến không âm
ta có thể đổi biến như sau Đặt p = a + b + c; q = ab + bc + ca; r = abc. Ta
có một số bất đẳng thức sau


• ab(a + b) + bc(b + c) + ca(c + a) = pq − 3r
• (a + b)(b + c)(c + a) = pq − r
• ab(a2 + b2 ) + bc(b2 + c2 ) + ca(c2 +2 ) = p2 q − 2q 2 − pr
• (a + b)(a + c) + (b + c)(b + a) + (c + a)(c + b) = p2 + q


2

+ b2 + c2 = p2 − 2q

• a3 + b3 + c3 = p3 − 3pq + 3r
• a4 + b4 + c4 = p4 − 4p2 q + 2q 2 + 4pr
• a2 b2 + b2 c2 + c2 a2 = q 2 − 2pr
• a3 b3 + b3 c3 + c3 a3 = q 3 − 3pqr + 3r2
• a4 b4 + b4 c4 + c4 a4 = q 4 − 4pq 2 r + 2p2 r2 + 4qr2
Có thể thấy ngay lợi ích của phương pháp này là mối ràng buộc giữa các
biến p, q, r mà các biến a, b, c ban đầu không có như

• p2 ≥ 3q
• p3 ≥ 27r
• q 2 ≥ 3pr
• pq ≥ 9r
• 2p3 + 9r ≥ 7pq
• p2 q + 3pr ≥ 4q 2
• p4 + 4q 2 + 6pr ≥ 5p2 q
p(4q − p2 )
• r≥
9
(4q − p2 )(p2 − q)
• r≥

6p


9

1.5

Vai trò của hàm số mũ, hàm logarit trong chứng
minh các bất đẳng thức cổ điển

Định lý 1.9 (Bất đẳng thức AM - GM suy rộng, [1]). Giả sử cho trước hai
cặp dãy số dương x1 , x2 , . . . , xn ; p1 , p2 , . . . , pn . Khi đó:

xp11

·

xp22

· · · xpnn ≤

x1 p1 + x2 p2 + · · ·xn pn
p1 + p2 + · · ·pn

p1 +p2 +···pn

.

Dấu đẳng thức xảy ra khi và chỉ khi x1 = x2 = · · · = xn .
Chứng minh. Đặt


s=

x1 p1 + x2 p2 + · · ·xn pn
.
p1 + p2 + · · ·pn

Sử dụng bất đẳng thức hàm mũ

ex−1 ≥ x, ∀x ∈ R,
ta thu được

Từ đó ta thu được hệ

Suy ra

x1
x1
x1
≤ e s −1 ⇔ x1 ≤ se s −1 .
s


x1
s −1 ,
x

se

1


x2

x2 ≤ se s −1 ,

···········


xn
xn ≤ se s −1 .
 p1
x
p1 ( s1 −1)p1
x

s
e
,


x
 1p2
p2 ( s2 −1)p2
x2 ≤ s e
,

···········

 pn
xn

xn ≤ spn e( s −n)pn .

Vậy nên

xp11 · xp22 · · · xpnn ≤ sp1 +p2 +···pn e

x1 p1 +x2 p2 +···+xn pn
−(p1 +p2 +···+pn )
s

hay

xp11 · xp22 · · · xpnn ≤ sp1 +p2 +···+pn , (đpcm).
x1
x2
xn
Dấu đẳng thức xảy ra khi và chỉ khi
=
= ··· =
= 1 hayx1 = x2 =
s
s
s
· · · = xn .


10

Định lý 1.10 (Bất đẳng thức dạng Katamata). Giả thiết cho ba bộ số dương


(αi ), (ui ), (xi ) thỏa mãn các điều kiện sau

u1 ≤ u2 ≤ · · · ≤ un ,




x1 α1 ≤ x1 u1 ,



x1 α1 + x2 α2 ≤ x1 u1 + x2 u2 ,
· · · · · · ··




x1 α1 + x2 α2 + · · · + xn−1 αn−1 ≤ x1 u1 + x2 u2 + · · · + xn−1 un−1 ,



x1 α1 + x2 α2 + · · · + xn αn = x1 u1 + x2 u2 + · · · + xn un .
Khi đó ta có

α1x1 α2x2 · · · αnxn ≤ ux1 1 ux2 2 · · · uxnn .
Dấu đẳng thức xảy ra khi và chỉ khi αi = ui , ∀i = 1, 2, . . . , n.
Chứng minh. Ta chứng minh bằng phương pháp quy nạp thông thường.
Với n = 2 tức là



u1 ≤ u2 ,
x1 α1 ≤ x1 u1 ,

x1 α1 + x2 α2 = x1 u1 + x2 u2 .

Ta chứng minh

α1x1 α2x2 ≤ ux1 1 ux2 2 .
Dấu đẳng thức xảy ra khi và chỉ khi α1 = u1 , α2 = u2 .
Xét hàm số

y=
Ta có

α
x1

y =

x1 −1

Vậy nên

y =0⇔
và y > 0 khi

x1

s−α
x2


s−α
x2
x2 −1

x2

.

s−α
α

.
x2
x1

α
s−α
s
=
=
,
x1
x2
x1 + x2

s
s
<
.

x1
x1 + x2

Do đó với

thì ta có

α
x1

α
u
s


,
x1
x1
x1 + x2
α
x1

x1

s−α
x2

x2




u
x1

x1

s−u
x2

x2

.


11

Đặt

α
s−α
u
s−u
= α1 ,
= α2 ,
= u1 ,
= u2 .
x1
x2
x1
x2


Suy ra, khi

α1 ≤ u1 ≤ u2 , x1 α1 + x2 α2 = x1 u1 + x2 u2 ,
ta thu được

α1x1 α2x2 ≤ ux1 1 ux2 2 .
Nhận thấy dấu đẳng thức xảy ra khi và chỉ khi αi = ui , ∀i = 1, 2.
Vậy định lý đúng với n = 2.
Với n = 3 tức là

u1 ≤ u2 ≤ u3 ,



x1 α1 ≤ x1 u1 ,
x1 α1 + x2 α2 ≤ x1 u1 + x2 u2 ,



x1 α1 + x2 α2 + x3 α3 = x1 u1 + x2 u2 + x3 u3 .
Ta chứng minh

α1x1 α2x2 α3x3 ≤ ux1 1 ux2 2 ux3 3 .
Dấu đẳng thức xảy ra khi và chỉ khi αi = ui , ∀i = 1, 2, 3.
Do giả thiết nên ta chỉ cần xét hai trường hợp
Trường hợp 1).
Khi



α1 = u1 − d1 ,
α2 = u2 − d2 ,

α3 = u3 + d3 .

Với d1 , d2 , d3 ≥ 0.
Khi đó x1 d1 + x2 d2 = x3 d3 và từ đó ta thu được

α1x1 α2x2 α3x3 = (u1 − d1 )x1 (u2 − d2 )x2 (u3 + d3 )x3
≤ ux1 1 (u2 − d2 )x2 u3 + d3 −

x1 d1
x3

x3

≤ ux1 1 ux2 2 ux3 3 .

Dấu đẳng thức xảy ra khi và chỉ khi αi = ui , ∀1 = 1, 2, 3.
Trường hợp 2).
Khi


α1 = u1 − d1 ,
α2 = u2 + d2 ,

α3 = u3 + d3 .


12


Với d1 , d2 , d3 ≥ 0.
Khi đó x1 d1 = x2 d2 + x3 d3 và từ đó ta thu được

α1x1 α2x2 α3x3 = (u1 − d1 )x1 (u2 + d2 )x2 (u3 + d3 )x3
x2 d2 x1 x2
u2 (u3 + d3 )x3 ≤ ux1 1 ux2 2 ux3 3 .
≤ u1 − d1 +
x1
Dấu đẳng thức xảy ra khi và chỉ khi αi = ui , ∀1 = 1, 2, 3.
Vậy định lý đúng với n = 3.
Giả sử định lý đúng với n. Ta chứng minh định lý đúng với n + 1.
Theo giả thiết thì

u1 ≤ u2 ≤ · · · ≤ un ,




x1 α1 ≤ x1 u1 ,



x1 α1 + x2 α2 ≤ x1 u1 + x2 u2 ,
· · · · · · ··




x1 α1 + x2 α2 + · · · + xn αn ≤ x1 u1 + x2 u2 + · · · + xn un ,




x1 α1 + x2 α2 + · · · + xn+1 αn+1 = x1 u1 + x2 u2 + · · · + xn+1 un+1 .
nên ứng với chỉ số k , ta có thể giả sử

αk ≤ uk , αk = uk − dk , uk+1 ≤ αk+1 , αk+1 = uk+1 + dk+1 .
Ta chia ra hai trường hợp để xét.
Trường hợp 1). Khi xk dk ≤ xk+1 dk+1 , thì
x

x

x

x

k−1 xk
k+1
k+2
n+1
α1x1 · · · αk−1
αk αk+1
αk+2
· · · αn+1

x

k−1
= α1x1 · · · αk−1

uk − dk

x

k−1 xk
≤ α1x1 · · · αk−1
uk

xk

uk+1 − dk+1
xk dk
uk+1 + dk+1 −
xk+1

xk+1
xk+1

x

x

x

x

k+2
n+1
αk+2
· · · αn+1


k+2
n+1
αk+2
· · · αn+1

x

n+1
≤ ux1 1 ux2 2 · · · un+1
.

Dấu đẳng thức xảy ra khi và chỉ khi αi = ui , với mọi i = 1, 2, . . . , n + 1.
Trường hợp 2). Khi xk dk ≥ xk+1 dk+1 , thì
x

x

x

x

k−1 xk
k+1
k+2
n+1
α1x1 · · · αk−1
αk αk+1
αk+2
· · · αn+1


x

k−1
= α1x1 · · · αk−1
uk − dk

xk

x

x

x

k+2
n+1
uk+1 − dk+1 k+1 αk+2
· · · αn+1
xk+1 dk+1 xk xk+1 xk+2
xn+1
xk−1
uk+1 αk+2 · · · αn+1
≤ α1x1 · · · αk−1
uk − dk +
xk
xn+1
x1 x2
≤ u1 u2 · · · un+1 .


Dấu đẳng thức xảy ra khi và chỉ khi αi = ui , với mọi i = 1, 2, . . . , n + 1.
Vậy định lý đúng với mọi n ≥ 2.


13

Chương 2
Các bất đẳng thức trong lớp hàm
mũ, hàm logarit
2.1

Bất đẳng thức hàm số mũ
Đối với hàm mũ, ta thường sử dụng kết quả sau để chứng minh bất đẳng

thức.
Định lý 2.1. Với a > 0; a = 1 thì hàm f (x) = ax luôn thỏa mãn bất đẳng
thức

f (x) ≥ f (x0 ) + f (x0 )(x − x0 ), ∀x, x0 ∈ R.

(2.1)

Chứng minh. Thật vậy
- Nếu x = x0 ta được đẳng thức.
- Xét x > x0 ta được khoảng (x0 ; x) và bất đẳng thức (2.1) có dạng

f (x) − f (x0 )
≥ f (x0 )
x − x0
hay f (x1 ) ≥ f (x0 ) với x0 < x1 < x.

Điều này là hiển nhiên vì hàm số f (x) = ax có f (x) = (ln a)2 .ax > 0 với
mọi a > 0, a = 1, x ∈ R nên f là hàm đơn điệu tăng trên R.
- Xét x < x0 ta được khoảng (x; x0 ) và bất đẳng thức (2.1) có dạng

f (x) − f (x0 )
≤ f (x0 )
x − x0
hay f (x1 ) ≤ f (x0 ) với x < x1 < x0 .
Điều này hiển nhiên vì f là hàm đơn điệu tăng trên R. Vậy ta thu được bất
đẳng thức (2.1), đpcm.
Từ kết quả của định lí 2.1 ta thu được kết quả của một số bài toán cực
trị trong lớp hàm mũ với tổng không đổi.


14

Hệ quả 2.1. Với a > 1 và x + y + z = α + β + γ thì hàm f (x) = ax luôn
thỏa mãn bất đẳng thức

f (y)
f (z)
f (α)
f (β)
f (γ)
f (x)
+
+

+
+

.
f (α) f (β) f (γ) f (α) f (β) f (γ)

(2.2)

Nói cách khác, với α, β, γ cho trước và x, y, z thay đổi thì giá trị nhỏ nhất
của biểu thức

M=


f (x)
f (y)
f (z)
+
+
f (α) f (β) f (γ)

f (α)
f (β)
f (γ)
+
+
.
f (α) f (β) f (γ)

Hệ quả 2.2. Với 0 < a < 1 và x + y + z = α + β + γ thì hàm f (x) = ax
luôn thỏa mãn bất đẳng thức

f (x)

f (y)
f (z)
f (α)
f (β)
f (γ)
+
+

+
+
.
f (α) f (β) f (γ) f (α) f (β) f (γ)

(2.3)

Nói cách khác, với α, β, γ cho trước và x, y, z thay đổi thì giá trị lớn nhất
của biểu thức

M=


f (x)
f (y)
f (z)
+
+
f (α) f (β) f (γ)

f (α)
f (β)

f (γ)
+
+
.
f (α) f (β) f (γ)

Bài toán 2.1. Cho x, y, z > 0. Chứng minh rằng

xx y y z z ≥ (xyz)

x+y+z
3

.

Chứng minh. Ta có hàm số y = ln t đồng biến trên khoảng (0; +∞) nên
ta có với mọi x, y, z > 0

(ln x − ln y)(x − y) ≥ 0 ⇔ xln x + y ln y ≥ xln y + y ln x,
(ln y − ln z)(y − z) ≥ 0 ⇔ y ln y + z ln z ≥ y ln z + z ln y,
(ln z − ln z)(z − x) ≥ 0 ⇔ z ln z + xln x ≥ z ln x + xln z.
Cộng theo về các bất đẳng thức trên ta được

2(xln x + y ln y + z ln z) ≥ (xln y + y ln x) + (y ln z + z ln y) + (z ln x + xln z)
⇔ 3(xln x + y ln y + z ln z) ≥ (x + y + z)(ln x + ln y + ln z)


15

⇔ 3(ln xx + ln y y + ln z z ) ≥ ln (xyz)x+y+z

⇔ ln (xx y y z z ) ≥ ln (xyz)
⇔ xx y y z z ≥ (xyz)

x+y+z
3

x+y+z
3

.

(Do hàm ln t đồng biến trên (0; +∞) ).
Bài toán 2.2. Cho x, y, z là độ dài 3 cạnh tam giác. Chứng minh

(x + y − z)x (y + z − x)y (z + x − y)z ≤ xx y y z z .
Chứng minh. Bất đẳng thức cần chứng minh tương đương với
y−z x
z−x y
x−y z
≤ 1.
1+
1+
1+
x
y
z
Vì x, y, z là độ dài ba cạnh tam giác nên x, y, z > 0 và

y−z


> 0,
1+



x

z−x
1+
> 0,

y



1 + x − y > 0.
z
Áp dụng bất đẳng thức AM - GM suy rộng ta có
y−z x
z−x y
x−y z
1+
1+
1+
x
y
z
z−x
x−y
y−z

+y 1 +
+z 1 +
x(1 +
x
y
z

x+y+z

=

x+y+z

x+y+z
x+y+z

Do đó

1+

y−z
x

x

x+y+z

= 1.

1+


z−x
y

y

1+

x−y z
≤ 1, (đpcm) .
z

Bài toán 2.3. Cho x, y, z > 0 thỏa mãn x2 + y 2 + z 2 = 1. Chứng minh
2

2

2

xx y y z z
2
2
2

≥ xy +1 y z +1 z x +1 .
3 3
Chứng minh. Bất đẳng thức cần chứng minh tương đương với

x2y


2

+z 2 2z 2 +x2 2x2 +y 2

y

z

1
≤ √ .
3 3


16

Để ý rằng

2y 2 + z 2 2z 2 + x2 2x2 + y 2
+
+
= 1.
3
3
3
Theo bất đẳng thức AM - GM suy rộng
x2y

2

+z


2

y 2z

2

+x

2

z 2x

2

+y

2

3



x(2y 2 + z 2 ) + y(2z 2 + x2 ) + z(2x2 + y 2 )
3

=

x(y 2 + 1 − x2 ) + y(z 2 + 1 − y 2 ) + z(x2 + 1 − z 2 )
3


Như vậy ta cần chứng minh

x(y 2 + 1 − x2 ) + y(z 2 + 1 − y 2 ) + z(x2 + 1 − z 2 ) ≤
hay
2

2

2

3

3

3

(xy + yz + zx ) + (x + y + z) − (x + y + z ) ≤





3

3.

Đặt p = x + y + z ; q = xy + yz + zx; r = xyz suy ra




3
0
<
p



01


0 < r ≤ √ .
3 3
giả thiết trở thành p2 − 2q = 1.
Sử dụng bất đẳng thức quen thuộc sau

xy 2 + yz 2 + zx2 + xyz ≤

4(x + y + z)3
27

hay

4
xy 2 + yz 2 + zx2 ≤ √ − r.
3 3
Ta thu được

4

V T (∗) ≤ √ − r + p − [3r + p(1 − q)]
3 3

4
= √ + pq − 4r .
3 3

Ta sẽ chứng minh


4
√ + pq − 4r ≤ 3.
3 3
Theo bất đẳng thức Schur bậc 3
r≥

p(4q − p2 )
9

=

p(2q − 1)
9

(∗)

3

.



17

nên bất đẳng thức cần chứng minh tương đương với
4p(2q − 1) √
4
√ + pq −
≤ 3
9
3 3
hay


pq + 4p ≤ 5 3.

Dễ thấy bất đẳng thức này luôn đúng do p ≤



3; q ≤ 1.
1
Dấu đẳng thức xảy ra khi và chỉ khi x = y = z = √ .
3

Bài toán 2.4. Cho x, y, z ∈ [0; 1]. Chứng minh

(2x + 2y + 2z )(2−x + 2−y + 2−z ) <

81
.

8

Chứng minh. Đặt 2x = a; 2y = b; 2z = c suy ra a, b, c ∈ [1; 2].
Ta có 1 ≤ a ≤ 2 suy ra

(a − 1)(a − 2) ≤ 0
⇔ a2 − 3a + 2 ≤ 0
2
⇔ a + ≤ 3.
a
Tương tự

2
≤ 3,
b
2
c + ≤ 3.
c
Cộng theo vế các bất đẳng thức trên ta có
b+

1 1 1 AM-GM
1 1 1
+ +
≥ 2 (a + b + c) · 2( + + )
a b c
a b c
81
1 1 1


≥ (a + b + c) + +
8
a b c

9≥a+b+c+2

hay

(2x + 2y + 2z )(2−x + 2−y + 2−z ) ≤

81
.
8

Dấu đẳng thức xảy ra khi và chỉ khi

a+b+c=2
a, b, c ∈ {1; 2}

1 1 1
+ +
a b c

hệ này vô nghiệm. Do đó dấu đẳng thức không xảy ra.


18

Bài toán 2.5. Cho x, y, z > 0. Chứng minh


xy+z + y z+x + z x+y > 1.
Nhận xét 2.1. Bài này có thể xem là ví dụ điển hình cho cách sử dụng bất
đẳng thức Bernoulli. Với tư tưởng này ta sẽ phải xét vị trí tương đối của các
tổng x + y ; y + z ; z + x so với 1.
Chứng minh. Dễ dàng nhận thấy rằng nếu hoặc x, hoặc y , hoặc z lớn hơn
hoặc bằng 1 thì bất đẳng thức hiển nhiên đúng.
Vậy ta sẽ xét x; y; z ∈ (0; 1). Không giảm tổng quát, giả sử x ≥ y ≥ z .
Do min{x + y; y + z; z + x} = y + z nên ta xét 2 trường hợp của y + z .
Trường hợp 1. y + z ≥ 1. Sử dụng bất đẳng tức Bernoulli ta có

xy+z = (1 + x − 1)y+z ≥ 1 + (x − 1)(y + z).
Tương tự

y z+x ≥ 1 + (y − 1)(z + x),
z x+y ≥ 1 + (z − 1)(x + y).
Như vậy

xy+z + y z+x + z x+y ≥ 3 + 2(xy + yz + zx − x − y − z).
Do x, y, z ∈ (0; 1) nên

(1 − x)(1 − y)(1 − z) > 0
⇔ 1 + xy + yz + zx − x − y − z > xyz > 0.
Do đó ta có

3 + 2(xy + yz + zx − x − y − z) > 1.
Vậy trong trường hợp này

xy+z + y z+x + z x+y > 1.
Trường hợp 2. y + z ≤ 1. Xét 3 trường hợp con
- Nếu y + z ≤ z + x ≤ x + y ≤ 1.

Sử dụng bất đẳng thức Bernoulli ta có

xy+z =

x
x1−(y+z)



x
1 + (x − 1)[1 − (y + z)]

=

x
.
x + y + z − x(y + z)


19

Suy ra

xy+z >
Tương tự

x
.
x+y+z


y
,
x+y+z
z
>
.
x+y+z

y z+x >
z x+y
Từ đó

xy+z + y z+x + z x+y > 1.
- Nếu y + z ≤ x + z ≤ 1 ≤ x + y Khi đó ta có

xy+z ≥ x; y z+x ≥ y
nên ta thu được

xy+z + y z+x + z x+y ≥ x + y + z x+y > 1.
-Nếu y + z ≤ 1 ≤ x + z ≤ x + y .
Lúc này ta có xy+z ≥ x.
Sử dụng bất đẳng thức Bernoulli ta có

y z+x ≥ 1 + (y − 1)(z + x),
z x+y ≥ 1 + (z − 1)(x + y).
suy ra

xy+z + y z+x + z x+y ≥ 2 + yz + (xy + yz + zx − x − y − z)
Do yz > 0; 1 + xy + yz + zx − x − y − z > 0 nên


2 + yz + (xy + yz + zx − x − y − z) > 1
Như vậy trong trường hợp này ta cũng có xy+z + y z+x + z x+y > 1.
Vậy bất đẳng thức đã được chứng minh.
Dấu đẳng thức xảy ra khi và chỉ khi (x; y; z) = (1; 0; 0) và các hoán vị.


20

Bài toán 2.6. Với x > 0, chứng minh rằng

2(1 + ex ) > (1 +



ex )2 >

(1 +


3

ex )3

2

.

Chứng minh. Xét hàm số f (t) = ln (1 + et ), mọi t > 0. Khi đó

et

f (t) =
> 0, ∀t > 0.
(1 + et )2
Vậy hàm f (t) là hàm lồi trên R+ . Mặt khác với x > 0 ta có

(x; 0; 0)

x x
; ;0
2 2

x x x
; ; .
3 3 3

Áp dụng bất đẳng thức Karamata đối với hàm lồi ta suy ra


3
ln (1 + ex ) + 2ln 2 > 2ln (1 + ex ) + ln 2 > 3ln (1 + ex )
3

3
x
1+ e

⇔ ln (1 + ex ) + ln 2 > ln (1 + ex )2 > ln
2
3


1 + 3 ex

⇔ 2(1 + ex ) > (1 + ex )2 >
.
2
Vậy bất đẳng thức đã được chứng minh.

Bài toán 2.7. Cho x, y ∈ N. Chứng minh rằng

x4 + y 4
x+y

3

x+y

≥ xx y y .

Chứng minh. Lấy Logarit nepe 2 vế ta có bất đẳng thức cần chứng minh
tương đương với

(x + y)ln
⇔ ln

3

3

x4 + y 4
≥ xln x + y ln y

x+y

x4 + y 4
x
y

ln x +
ln y.
x+y
x+y
x+y

Áp dụng bất đẳng thức Jensen ta có

x
y
x2 + y 2
ln x +
ln y ≤ ln
.
x+y
x+y
x+y


21

Như vậy ta cần chứng minh
4
4

x2 + y 2
3 x + y
≤ ln
ln
x+y
x+y
x2 + y 2 3 x4 + y 4


x+y
x+y
3

⇔ x2 + y 2 ≤ x + y

2

x4 + y 4

⇔ x6 + 3x4 y 2 + 3x2 y 4 + y 6 ≤ x6 + x2 y 4 + 2x5 y + 2xy 5 + y 2 x4 + y 6
⇔ x4 y 2 + x2 y 4 ≤ x5 y + xy 5
⇔ x2 y 2 (x2 + y 2 ) ≤ xy(x4 + y 4 )
⇔ x3 y + xy 3 ≤ x4 + y 4
⇔ x3 y − x4 + xy 3 − y 4 ≤ 0
⇔ x3 (y − x) + y 3 (x − y) ≤ 0
⇔ (y − x)(x3 − y 3 ) ≤ 0
⇔ −(x − y)2 (x2 + xy + y 2 ) ≤ 0.
Bất đẳng thức cuối luôn đúng do đó ta có bất đẳng thức phải chứng minh.

Bài toán 2.8. Chứng minh rằng với mọi số thực dương x, y, z thỏa mãn


x + y + z = 1 ta có
1+

1
x

y

1+

1
y

z

1+

1
z

x

≥1+

Chứng minh. Xét hàm số f (t) = ln 1 +

f (t) =
f (t) =


1
.
xy + yz + zx

1
; ∀t > 0
t

−1
t2 + t

2t + 1
>0
[t(t + 1)]2

suy ra hàm số lồi
Theo bất đẳng thức Jensen ta có

yf (x) + zf (y) + xf (z) ≥ f (xy + yz + zx)
1
1
1
1
⇔ y ln 1 + +z ln 1 + +xln 1 + ≥ ln 1 +
x
y
z
xy + yz + zx



22

⇔ 1+

1
x

y

1+

1
y

z

1+

1
z

x

≥1+

1
.
xy + yz + zx

Bài toán 2.9. Cho a1 , a2 , . . . , an > 0; x ≥ 1. Chứng minh rằng


a1 + a2 + · · · + an
ax1 + ax2 + · · · + axn

n
n

x

Chứng minh. Bất đẳng thức cần chứng minh tương đương với

a1 n
n

x

x

a2 n

+

+···+

n

ai

an n
n


ai

i=1

x

≥ n.

ai

i=1

i=1

Áp dụng bất đẳng thức Bernoulli ta có

na1
a1 + a2 + · · · + an

x

(n − 1)a1 − (a2 + a3 + · · · + an ) x
a1 + a2 + · · · + an
(n − 1)a1 − (a2 + a3 + · · · + an )
≥ x.
+ 1.
a1 + a2 + · · · + an
= 1+


Chứng minh tương tự ta cũng có

(n − 1)a2 − (a1 + a3 + · · · + an ) x
a1 + a2 + · · · + an
(n − 1)a2 − (a1 + a3 + · · · + an )
≥ x.
+1
a1 + a2 + · · · + an
· · · · · · · · · · · · · · · · · · · · · · · · ··
x
(n − 1)an − (a1 + a3 + · · · + an−1 )
nan
≥ x.
+ 1.
a1 + a2 + · · · + an−1
a1 + a2 + · · · + an
na2
a1 + a2 + · · · + an

x

= 1+

Cộng theo vế các bất đẳng thức trên ta có bất đẳng thức cần chứng minh.

Bài toán 2.10. Cho các số dương x1 , x2 , . . . , xn và y1 , y2 , . . . , yn . Chứng
minh rằng

x1 + x2 + · · · + xn
y1 + y2 + · · · + yn


y1 +y2 +···yn

Chứng minh.
Xét hàm số f (t) = ln t có f (t) =



x1
y1

y1

x2
y2

y1

···

xn
yn

yn

.

−1
< 0 suy ra f (t) là hàm số lõm.
t2


Áp dụng bất đẳng thức Jensen

u1 f (t1 ) + u2 f (t2 ) + · · · + un f (tn ) ≤ f (u1 t1 + u2 t2 + · · · + un tn )


23

với u1 + u2 + · · · + un = 1 ta được

y1

x1
x2
xn
y2
y1
+ n f
+···+ n f
y1
y2
yn
yi
yi

f

n

yi

i=1

≤f

i=1

y1

·

n

yi

x1
yn x n
·
+···+ n
y1
y
yi n

i+1

=f

i=1

i+1


x1 + x2 + · · · + xn
y1 + y 2 + · · · + y n

x1
x2
xn
+y2 f
+ · · · +yn f
y1
y2
yn
x1 + x2 + · · · + xn
≤ (y1 + y2 + · · · + yn )f
y1 + y2 + · · · + yn
x1
x2
xn
⇔ y1 ln
+y2 ln
+ · · · +yn ln
y1
y2
yn
x1 + · · · + xn
≤ (y1 + y2 + · · · + yn )ln
y1 + · · · + yn
x1 y1 x2 y2
xn yn

···

y1
y2
yn
x1 + x2 + · · · + xn y1 +y2 +···+yn

.
y1 + y2 + · · · + yn
⇔ y1 f

Vậy bất đẳng thức đã được chứng minh.

Bài toán 2.11. Cho 0 < x ≤ y ≤ 4 và

1 1
+ ≥ 1. Chứng minh rằng
x y

xy ≤ y x .
Chứng minh. Áp dụng bất đẳng thức Bernoulli ta có

1
x

y
x

y
x2 + y − xy
y 1
.

≥ . +1− =
x x
x
x2



x2
+1−x
y
x2
y
≥y
+ 1 − x = (x − 2)2 ≥ 0
4
4

x2 + y − xy = y


24

nên từ trên ta có

x2
x ≤ 2
.
x + y − xy
y
x


Do đó ta cần chứng minh

x2
≤y
x2 + y − xy
⇔ x2 ≤ x2 y + y 2 − xy 2
⇔ x2 − y 2 ≤ x2 y − xy 2
⇔ (x − y)(x + y) ≤ xy(x − y)
⇔ (y + x − xy)(x − y) ≤ 0
1 1
+ − 1 (x − y) ≤ 0.

x y
Điều này luôn đúng theo giả thiết. Từ đây suy ra
y

xx ≤ y
hay

xy ≤ y x .
Dấu đẳng thức xảy ra khi và chỉ khi x = y .

Bài toán 2.12. Cho x, y, z > 0. Chứng minh

(y + z)x + (z + x)y + (x + y)z > 2.
Lời giải. Ta xét các trường hợp sau
- Trường hợp 1. Trong 3 số x, y, z có ít nhất 2 số ≥ 1, chẳng hạn x, y ≥ 1.
Lúc đó


(y + z)x + (z + x)y + (x + y)z > 1 + 1 + (x + y)z > 2.
- Trường hợp 2. Trong 3 số x, y, z có duy nhất 1 số ≥ 1, chẳng hạn x ≥ 1;

0 < y, z ≤ 1.
Khi đó. Áp dụng bất đẳng thức Bernoulli ta có
1
1
≤1+y
− 1 < 1 (Vì x + z > 1)
y
(z + x)
z+x


×